Hey, isn't that my birth year?

Algebra Level 5

Find the sum of all roots to the equation below, given that there are no multiple roots.

x 2001 + ( 1 2 x ) 2001 = 0 x^{2001} + \left(\dfrac{1}{2} - x\right )^{2001} = 0


The answer is 500.

This section requires Javascript.
You are seeing this because something didn't load right. We suggest you, (a) try refreshing the page, (b) enabling javascript if it is disabled on your browser and, finally, (c) loading the non-javascript version of this page . We're sorry about the hassle.

4 solutions

Otto Bretscher
Mar 17, 2016

If a a is a root, then so is 1 2 a \frac{1}{2}-a ; plug it in! Thus the average value of the roots is 1 4 \frac{1}{4} . This being a polynomial of degree 2000 (the x 2001 x^{2001} term cancels out), the sum of the roots is 2000 × 1 4 = 500 2000\times \frac{1}{4}=\boxed{500}

What a great insight!

Saurabh Chaturvedi - 5 years, 2 months ago

Woahhhh! This is nice!

Pi Han Goh - 5 years, 2 months ago

definitely beautiful

Joel Yip - 5 years, 2 months ago

I have even applauded to this solution. So nicely done!

Vitalii Feilo - 5 years, 2 months ago

Done the same way

Ravneet Singh - 5 years, 2 months ago
Rishabh Jain
Mar 17, 2016

Expand ( x + 1 2 ) 2001 \left(-x+\dfrac{1}{2} \right )^{2001} using Binomial Expansion : x 2001 + ( ( 2001 0 ) x 2001 + ( 2001 1 ) x 2000 × 1 2 ( 2001 2 ) x 1999 × 1 4 ) = 0 \cancel{x^{2001}} + \left(\cancel{-\binom{2001}{0}x^{2001}}+\binom{2001}{1}x^{2000}\times \dfrac 12 -\binom{2001}{2}x^{1999}\times \dfrac 14\cdots\right)=0 ( 2001 1 ) x 2000 × 1 2 ( 2001 2 ) x 1999 × 1 4 = 0 \implies\binom{2001}{1}x^{2000}\times \dfrac 12 -\binom{2001}{2}x^{1999}\times \dfrac 14\cdots=0 By Vieta's formula . sum of roots of this equation are given by:-

( 2001 2 ) × 1 4 ( 2001 1 ) × 1 2 = 500 \dfrac{\binom{2001}{2}\times \dfrac 14}{\binom{2001}{1}\times \dfrac 12}=\boxed{500}

Dude, please let us add some/few solutions. _/ _

Akhil Bansal - 5 years, 3 months ago

Log in to reply

Yes please -_-

Rishabh Jain - 5 years, 3 months ago

You need to be very active to add solutions :P

Rohit Udaiwal - 5 years, 2 months ago

But my doubt is is anish so much old?? Same process was mine so I replied here.

Shyambhu Mukherjee - 5 years, 2 months ago

Log in to reply

I only attempted because I am as old as Anish...

Margaret Zheng - 5 years, 2 months ago
Anthony Ritz
Mar 20, 2016

See also this basically identical question and associated solutions: Will you really find roots?

Aakash Khandelwal
Mar 20, 2016

Can any one explain why this is level 4

5 now, for some reason.

A Former Brilliant Member - 5 years, 2 months ago

0 pending reports

×

Problem Loading...

Note Loading...

Set Loading...